Simplify each

2. 2
— - —
5 3


3 3
— + —
4 8
With explanation please

Answers

Answer 1

Answer:

1) [tex] - \frac{4}{5} [/tex]

2) [tex] \frac{9}{8} [/tex]

Step-by-step explanation:

[tex] \frac{2}{5} - \frac{2}{3} = \frac{2 \times 3}{5 \times 3} = \frac{2 \times 5}{3 \times 5} = \frac{6}{15} - \frac{10}{15} [/tex]

[tex] \frac{6 - 10}{15} = - (10 - 6) = - 4[/tex]

[tex] = - \frac{4}{15} [/tex]

°■°■°■°■°■°■°■°■°■°

[tex] \frac{3}{4}+\frac{3}{8} [/tex]

[tex] \frac{6+3}{8} [/tex]

[tex] = \frac{9}{8} [/tex]


Related Questions

l find it difficult to solve this problem. Please help.​

Answers

Answer:

27 square are there in the given figure

36 square are in the picture

Determine if the expression 8a – a4 is a polynomial or not. If it is a polynomial,
state the type and degree of the polynomial.
The given expression
va polynomial.

represents
does NOT
represent
attempt 1 out of 2 / problem 1 out of max 1

Answers

Answer:

Represents

Step-by-step explanation:

Going over the basic definition of polynomial, an expression which is composed of variables, constants and exponents, that are combined using the mathematical operations such as addition, subtraction, multiplication and division, we can see that the expression is indeed a polynomial.

SOMEONE HELP PLEASE IT'S DUE IN A FEW MINUTES

if f(x)=3x and g(x)=x+2 find (f-g) (x)

Answers

(f-g)(x) = f(x) - g(x)

= (3x) - (x+2)

= 3x - x - 2

= 2x - 2

Evaluate k + 3 when: *
4 points
51 13 15 43 9 45 37
k = 12
k = 48
k = 10
k = 40
k = 12
k = 48
k = 10
k = 40

Answers

Step-by-step explanation:

12 + 3=15

48+ 3=51

10 + 3=13

40 + 3=43

12 + 3=15

48 + 3=51

10 + 3=13

40+ 3=43

4/9 and 10/3 least common denominator?

Answers

Answer:

4/9 and 30/9

Step-by-step explanation:

9, 18, 27, 36

3, 6, 9, 12,

4/9 already has 9 denominator

10/3 × 3/3= 30/9

so now both numbers have common denominators.

4/9 and 30/9

4/9 and 30/9
I did the work

order from least to greatest 4^2,√15 ,-5/2,-2 1/3,1/16

Answers

Answer:

-2 1/3, 1/16, √15, 4^2

Following order from least to greatest.

can any one solve this please....I kinda need this​

Answers

9514 1404 393

Answer:

t₂ = 2t₃ = 2

Step-by-step explanation:

The given expression says ...

  [tex]t_n+1=3\\\\t_n=2\qquad\text{for any value of n}\\\\t_2=2\\\\t_3=2[/tex]

Answer:

[tex]here \: tn + 1 = 3 \\ tn = 2 \\ put \: n = 1 \\ t1 = 2 \\ n = 2 \\ t2 = 2 \\ n = 3 \\ t3 = 2 \\ for \: all \: n \: values \: = 2 \\ thank \: you[/tex]

can someone please help

5(1+2m)=1/2(8+20m)

Answers

Answer:

no solutions

Step-by-step explanation:

5(1+2m)=1/2(8+20m)

5+10m = 4+10m

10m-10m= 4-5

No solutions

Jaulas father throws Kayla a graduation party that costs $773. He pays the DJ $250, $75 for the cake, and $4 per party guest for food. How many guests were at the party?

Answers

Answer:

112 guests

Step-by-step explanation:

I. If her total cost is 773$

When Total = DJ + Cake + Guest

            773 = 250 + 75 + 4(x)

            773 = 325 + 4(x)

            773 - 325 = 4(x)

            448 = 4x

               x = 448/4

               x = 112

That makes guests are 112

Marcus is tying two bows for birthday gifts. He uses 3 3/8 feet of ribbon for Crystal's gift and 2 3/4 feet for Tim's gift. If he has an 8-foot roll of ribbon, how much will Marcus have left over?​

Answers

Marcus will have [tex]1\frac{7}{8}[/tex] feet as left-over.

It is important we understand that foot refers to single unit of measurement while feet is the plural of foot.

Marcus needs to tie two bows for birthday gifts and he has an 8-foot roll of ribbon.

• He uses 3 3/8 (27/8 as a fraction) feet of ribbon for Crystal's gift.

• He uses 2 3/4 (11/4 as a fraction) feet for Tim's gift.

The left-over computation is as follows:

= [tex]8 - \frac{27}{8} - \frac{11}4}[/tex]

= [tex]\frac{256-108-88}{32}[/tex]

= [tex]\frac{60}{32}[/tex]

= [tex]\frac{15}{8}[/tex] or [tex]1\frac{7}{8}[/tex] feet

Marcus will have [tex]1\frac{7}{8}[/tex] feet as left-over.

See related question here brainly.com/question/14702678

what is the median of these numbers? 4, 14, 7, 8, 12, 12, 5, 10​

Answers

The median of that data set is 9.

Sort the numbers from least to greatest and find the middle number (that should be your median)

If there are 2 numbers in the middle of the set, then you take the average and that would be your median.

Answer:

9

Step-by-step explanation:

4, 14, 7, 8, 12, 12, 5, 10

Step 1. Sort the numbers in order from least to greatest

4, 5, 7, 8, 10, 12, 12, 14

Step 2. Find the median (middle number)

Since the median is between 8 and 10, add the two numbers together and then divide sum by 2

8 + 10 = 18/2 = 9

How many sides does a rhombus HAVE? ​

Answers

a rhombus has about 4 sides

Can someone give me a simple explanation of how to calculate probability? Thanks!

Answers

Step-by-step explanation:

Determine a single event with a single outcome. ...Identify the total number of outcomes that can occur. ...Divide the number of events by the number of possible outcomes. ...Determine each event you will calculate. ...Calculate the probability of each event.

pls mark me the brainliest pls

what they said i guess, i jus need points so i can get answers ya feel

Write –0.4 in simplest form.

Answers

Answer as a Fraction: -2/5

Step-by-step explanation:

-0.4 = -4/10

       = -2/5

0.736euros= 1us dollar

Answers

Answer:

0.86 United States Dollar

Step-by-step explanation:

What is the slope of the line for BOTH graphs? Are the slopes the same or different?

PLS HELP MEEE

Answers

Answer:

both are -10

Step-by-step explanation:

#1

x1 y1  x2 y2

1.5 180  6 135

(Y2-Y1) (135)-(180)=   -45  ΔY -45

(X2-X1) (6)-(1.5)=    4.5  ΔX 4.5

slope= -10          

B= 195          

Y =-10X +195

~~~~~~~~~~~~~~~~~~~~~~~~~~~~~~~~    

#2

x1 y1  x2 y2

1 175  8 105

   

(Y2-Y1) (105)-(175)=   -70  ΔY -70

(X2-X1) (8)-(1)=    7  ΔX 7

slope= -10          

B= 185          

Y =-10X +185    

What type of number is 80i?

Answers

Imaginary number because of the i !!!

(3/5) + (-6/7)+(-2/5)+(3/14) find​

Answers

Answer:

Step-by-step explanation:

Rearrange the givens

3/5 - 2/5 - 6/7 + 3/14

1/5 - 12/14 + 3/14

1/5 -  9/14

The lowest common multiple is 70

14/70 - 45/70 = - 31/70

Evaluate 3yz +20 -xy if x =2, y = -3, and z = 4

Answers

Answer:

Ans. is -10. Explanation is in the attachment.

ATTENDING TO PRECISION The diagram shows an insect called a walking stick Use the ruler to estimate the length of the abdomen and the length of the thorax to the nearest inch. How much longer is the walking stick's abdomen than its thorax? How many times longer is its abdomen than its thorax?​

Answers

Estimation and precision are used when the actual value cannot be easily determined or calculated.

The length of the abdomen is [tex]2\frac 14[/tex] inchesThe length of the thorax is [tex]1\frac 34[/tex] inchesThe abdomen is longer than the thorax [tex]1\frac 27[/tex] times

Given that, the length is to be measured to the nearest [tex]\frac 14\ inch[/tex], we have the following observations

The length of the abdomen starts at the 0 mark and ends after the 2-mark. The length ends before [tex]2\frac{1}{2}[/tex]. So, the length of the abdomen can be estimated to [tex]2\frac{1}{4}[/tex] --- to the nearest [tex]\frac{1}{4}[/tex].Similarly, the length of the thorax starts at the [tex]2\frac{1}{4}[/tex] mark and ends before [tex]4\frac{1}{4}[/tex]. The end of the thorax length can be estimated to: [tex]4[/tex] --- to the nearest [tex]\frac{1}{4}[/tex].

So, we have:

[tex]L_1 = 2\frac 14[/tex] --- the length of the abdomen

[tex]L_2 = 4 - 2\frac 14[/tex]

Express as improper fraction

[tex]L_2 = 4 - \frac{9}{4}[/tex]

Take LCM

[tex]L_2 = \frac{16 - 9}{4}[/tex]

[tex]L_2 = \frac{7}{4}[/tex]

Express as proper fraction

[tex]L_2 = 1\frac{3}{4}[/tex] --- the length of the thorax

The number of times (n) the abdomen is longer than the thorax is calculated as follows:

[tex]n = L_1 \div L_2[/tex]

So, we have:

[tex]n = 2\frac 14 \div 1\frac 34[/tex]

Express as improper fractions

[tex]n = \frac 94 \div \frac 74[/tex]

Rewrite as:

[tex]n = \frac 94 \times \frac 47[/tex]

[tex]n = \frac 97[/tex]

Express as proper fractions

[tex]n = 1\frac 27[/tex]

Hence, the abdomen is [tex]1\frac 27[/tex] times longer than the thorax

Read more about estimation and precision at:

https://brainly.com/question/509148

One-fourth of a number decreased by 12 is 6. Find the number.​

Answers

Answer: 1/4n-12=6

1n/4-12=6

n=72

Step-by-step explanation:

Answer:

72

Step-by-step explanation:

let the number be n , then

[tex]\frac{1}{4}[/tex] n - 12 = 6 ( multiply through by 4 to eliminate the fraction )

n - 48 = 24 ( add 48 to both sides )

n = 72

solve for w 3w + 6w - 19 = -91
please

Answers

Hi ;-)

[tex]3w+6w-19=-91\\\\3w+6w=-91+19\\\\9w=-72 \ \ /:9\\\\\huge\boxed{x=-8}[/tex]

Answer:

w = -8

Step-by-step explanation:

Given:

3w + 6w - 19 = -91

Isolate w, add 19 to both sides:

3w + 6w = -91 + 19

Simplify both sides:

9w = -72

Divide both sides by 9:

w = -72/9w = -8

Write the conjugate of the following complex
number:
-6i

Answers

Answer:

[tex]6i[/tex]

Step-by-step explanation:

We want to write the conjugate of the complex number:

[tex]-6i[/tex]

Recall that the conjugate of a complex number in the form a + bi is a - bi, and vice versa.

We can rewrite our complex number as:

[tex]\displaystyle 0 - 6i[/tex]

Hence, a = 0 and b = 6.

Then its conjugate will be:

[tex]0 + 6i = 6i[/tex]

Round your answer to the nearest hundredth​

Answers

Answer:

200

Step-by-step explanation:

90-35=125

180-125=55

55x4=220

220= 200

6+m/2 = -3
Help me pleAseeee

Answers

Answer:

m = -18

Step-by-step explanation:

6 + m/2 = -3

Subtract 6 from each side

6 + m/2 -6 = -3-6

m/2 = -9

Multiply each side by 2

m/2 * 2 = -9*2

m = -18

[tex]\huge\text{Hey there!}[/tex]

[tex]\huge\boxed{\mathsf{6+ \dfrac{m}{2}=-3}}}\\\huge\boxed{\rightarrow\mathsf{6 + \dfrac{1}{2}m = -3}}[/tex]

[tex]\huge\boxed{\rightarrow \mathsf{\dfrac{1}{2}m+ 6 = -3}}\\\\\huge\boxed{\text{SUBTRACT 6 to BOTH SIDES}}\\\huge\boxed{\mathsf{\dfrac{1}{2}m + 6 - 6 = -3 + 3}}[/tex]

[tex]\huge\boxed{\mathsf{\dfrac{1}{2}m = -9}}[/tex]

[tex]\huge\boxed{\text{MULTIPLY 2 to BOTH SIDES}}[/tex]

[tex]\huge\boxed{\mathsf{\dfrac{1}{2}m \times 2 = 2 \times -9}}[/tex]

[tex]\huge\boxed{\mathsf{m = 2\times-9}}\\\huge\boxed{\mathsf{2\times-9 = m}}\\\huge\boxed{\mathsf{2\times-9 = \bf -18}}\\\\\huge\boxed{\textsf{Therefore, your answer: \bf -18}}\huge\checkmark[/tex]

[tex]\huge\text{Good luck on your assignment \& enjoy your day!}[/tex]

~[tex]\huge\boxed{\frak{Amphitrite1040:)}}[/tex]

Solve using Quadratic Formula
x^2 + 3x - 3 = 0​

Answers

Answer:

a= 1 , b= 3 , c= –3

[tex]x = \frac{ - b \frac{ + }{} \sqrt{ {b}^{2} - 4ac} }{2a} \\ x = \frac{ - 3 \frac{ + }{} {} \sqrt{ {3}^{2} - 4(1)( - 3)} }{2(1)} \\ = \frac{ - 3 \frac{ + }{} \sqrt{9 + 12} }{2} = \frac{ - 3 \frac{ + }{} \sqrt{21} }{2} \\ \\ x = \frac{ - 3 \frac{ + }{} \sqrt{21} }{2} \\ \\ x = 0.79 \\ x = - 3.79[/tex]

I hope I helped you^_^

(
a. Find the distance around the park to the nearest yard.
39
The distance is about
yards.
b. A new path and a bridge are constructed from point Q to the midpoint M of PR Find QM to the nearest yard.
QM
yd
c. A man jogs from P to Q to M to R to Q and back to P at an average speed of 150 yards per minute. To the neare
long does it take him to travel the entire distance?
It takes about
minutes
Type here to con

Answers

We have that the distance around the park to the nearest yard,QM to the nearest yard and the Time of travel for man's jog is given respectively as

[tex]X=190yd[/tex][tex]QM=40yd[/tex][tex]T=1.5minutes.[/tex]

From the Question we are told that

[tex]PQ=(50-10)=40\\\\QR=(80-10)70[/tex]

Generally the equation for the Resultant PR is mathematically given as

[tex]PR=\sqrt{PQ^2+QR^2}\\\\PR=\sqrt{70^2+40^2}\\\\PR=80yd[/tex]

a)

Generally the equation for the Distance around they park is mathematically given as

[tex]X=PQ+QR+PR\\\\X=80+70+40\\\\[/tex]

[tex]X=190yd[/tex]

b)

Generally the equation for the Path QM is mathematically given as

From the diagram we see that

[tex]QM=PM=MR\\\\QM=\frac{80}{2}\\\\[/tex]

[tex]QM=40yd[/tex]

c)

From the question we are told

Average speed of [tex]150yd/m[/tex]

Total distance traveled

[tex]D_t=40+40+40+70+40\\\\D_t=230yd[/tex]

Therefore

Time

[tex]T=\frac{D_t}{V}\\\\T=\frac{230}{150}[/tex]

[tex]T=1.5minutes.[/tex]

in conclusion

The distance around the park to the nearest yard,QM to the nearest yard and the Time of travel for man's jog is given respectively as

[tex]X=190yd[/tex][tex]QM=40yd[/tex][tex]T=1.5minutes.[/tex]

For more information on this visit

https://brainly.com/question/15356513

Marking brainliest

Don’t know what to write for the last 2

Answers

Answer:

Business, management, marketing and technology.

Engineering, manufacturing and industrial technology.

i am not sure but i hope it will help you.

please mark me as brainlest.

You are trying to make a hole in one on the miniature golf green
3
3
5
7
9 X
a. Write an absolute value function that represents the path of the golf ball.
3
$ (z) = – 31x/+6
b. Write the function in part (a) as a piecewise function.
f(x) =
if x < 6
if x26

Answers

a. f(x) = |4 - 2x/3|. You could write abs(4 - 2x/3)

b. f(x) = 2x/3 if x < 6
f(x) = 8 - 2x/3 if 6 <= x <= 9

2 1/6 x 4 1/2= and explain it

Answers

Answer:

9 3/4

Step-by-step explanation:

First you would want to multiply the whole number with their denominators and then add the numerator to that. 2*6+1 and 4*2+1  Then you keep the original denominator at the bottom of the fraction.

2*6+1/6  4*2+1/2 ---> 13/6 x 9/2

You would then simplify the crossing numbers 6 and 9. They both can be divided by three.

13/2 x 3/2 = 39/4

Now you would do long division where you divide 39 by 4

4 goes into 39 nine times subtracting 36 from 39.

You then end up with 1 which becomes you numerator, 4 is the new denominator and 9 is the whole number.

The answer of the given mixed numbers 2 1/6 x 4 1/2 is 9 3/4.

What is mixed numbers?

A mixed number is a whole number, and a proper fraction represented together. It generally represents a number between any two whole numbers.

You should first multiply the entire number by the denominators of the other numbers, and then add the numerator to that result. 2*6+1 and 4*2+1 The original denominator is then kept at the bottom of the fraction.

(2*6+1)/6 x ( 4*2+1)/2 —-> 13/6 x 9/2

Next, you would streamline the intersection of numbers 6 and 9. They are both divisible by three.

13/2 x 3/2 = 39/4

You would now perform a long division by dividing 39 by 4.
4 multiplies 39 by 9 and subtracts 36 from 39.

In the end 3 becomes the numerator, 4 becomes the new denominator, and 9 is the total.

Therefore, 2 1/6 x 4 1/2 = 9 3/4.

To know more about the mixed numbers, click on the link

https://brainly.com/question/1746829

#SPJ2

Other Questions
Rewrite each angle in radian measure as a multiple of .540Rewrite each angle in degree measure.-4 Someone help please 1= 39 degrees 2: 40 degrees 3: 41 degrees 4: 42 degrees Who helped Hernn Corts and his soldiers defeat the Aztec?AtahualpaFrancisco Pizarrothe PortugueseAmerican Indian warriors Menerangkan warisan seni bina rom di Malaysia. 1..? ESSENTIAL QUESTIONHow are themidpoint and length of a segment on thecoordinate plane determined? CORRECT ANSWER ONLY PLEASE!!! The table of values represents the function g(x) and the graph shows the function f(x). Using the following information: The bank statement balance is $5,086. The cash account balance is $5,263. Outstanding checks amounted to $894. Deposits in transit are $1,007. The bank service charge is $55. A check for $51 for supplies was recorded as $42 in the ledger. Prepare a bank reconciliation for Candace Co. for May 31. The most suitable unit for expressing nuclear radius isA) micro B) nanometreC) fermi D) angstrom how many grams of CaCo3, will react with dil HCL to produce 75gm of Co2 ? Which of the following choices BEST describes the relationship between intensity and time in a successfulexercise program?A as intensity increases, time should increaseB. as intensity decreases, time should decreaseC. as intensity increases, time should decreaseD. intensity and time should never changePlease select the best answer from the choices provided.O AOBOCODSave and ExitNextSubmit ASAP. Five stars + brainless.A procedure calls for 4 parts reagent A to 16 parts water.What is the dilution of reagent A in water?a) 4/16b) 1/5c) 1/4 d) 4/12 The set S = {x\-4 Plz help ASAP What happened in the first day of school. Tell what occurred that stands out in your memory. The Scarlet LetterBased on the commentary, what is the importance of the scaffold in the novel? Find the mean, median, and mode of 14, 15, 3, 15, 14, 14, 18, 15, 8, 16. Having media literacy enables you to do what?A. Make money by advertising your ideasB. Use different tools of media productionC. Figure out what products to buyD. Become a "critical consumer" about the information you receiveSUBMIT what is an expression for...4x + 4x + 4x + 4x + 4x + 4x If the toy car weighs 250g and has a kinetic energy of 2.0 J, what is its velocity 3. When six is added to six times a number the result obtained is the same as when the number is added to 10 andthe result multiplied by 3. Form an equation and solve it to find the unknown number. Which isotope of iron from the periodic table is most abundant